In $x^{x^{x^{x^{…}}}}=2$, the answer is $x=sqrt{2}$. Why not also $-sqrt{2}$?












1














From this question



$$x^{x^{x^{x^{...}}}}=2$$



The answer is clearly $sqrt{2}$, but I'm curious why it's not also $-sqrt{2}$.



Am I missing something basic?










share|cite|improve this question









New contributor




b.ben is a new contributor to this site. Take care in asking for clarification, commenting, and answering.
Check out our Code of Conduct.
















  • 5




    A negative non-integer to a negative non-integer power is a little bit problematic (in real numbers) ...
    – Matti P.
    Jan 4 at 9:07






  • 1




    By the definition, if we wrote $x^x$ then the domain is $(0,+infty).$
    – Michael Rozenberg
    Jan 4 at 9:07












  • @MichaelRozenberg though if you want $x^{x^{x^{x^{...}}}}$ to converge then you need $e^{-e} leq x leq e^{1/e}$ roughly $0.066 lt x lt 1.44$
    – Henry
    Jan 4 at 9:14










  • @Henry I said about domain only.
    – Michael Rozenberg
    Jan 4 at 9:15










  • The domain of $y = x^x$ is $x > 0$ if you’re dealing with only real numbers.
    – KM101
    Jan 4 at 9:15


















1














From this question



$$x^{x^{x^{x^{...}}}}=2$$



The answer is clearly $sqrt{2}$, but I'm curious why it's not also $-sqrt{2}$.



Am I missing something basic?










share|cite|improve this question









New contributor




b.ben is a new contributor to this site. Take care in asking for clarification, commenting, and answering.
Check out our Code of Conduct.
















  • 5




    A negative non-integer to a negative non-integer power is a little bit problematic (in real numbers) ...
    – Matti P.
    Jan 4 at 9:07






  • 1




    By the definition, if we wrote $x^x$ then the domain is $(0,+infty).$
    – Michael Rozenberg
    Jan 4 at 9:07












  • @MichaelRozenberg though if you want $x^{x^{x^{x^{...}}}}$ to converge then you need $e^{-e} leq x leq e^{1/e}$ roughly $0.066 lt x lt 1.44$
    – Henry
    Jan 4 at 9:14










  • @Henry I said about domain only.
    – Michael Rozenberg
    Jan 4 at 9:15










  • The domain of $y = x^x$ is $x > 0$ if you’re dealing with only real numbers.
    – KM101
    Jan 4 at 9:15
















1












1








1


1





From this question



$$x^{x^{x^{x^{...}}}}=2$$



The answer is clearly $sqrt{2}$, but I'm curious why it's not also $-sqrt{2}$.



Am I missing something basic?










share|cite|improve this question









New contributor




b.ben is a new contributor to this site. Take care in asking for clarification, commenting, and answering.
Check out our Code of Conduct.











From this question



$$x^{x^{x^{x^{...}}}}=2$$



The answer is clearly $sqrt{2}$, but I'm curious why it's not also $-sqrt{2}$.



Am I missing something basic?







puzzle






share|cite|improve this question









New contributor




b.ben is a new contributor to this site. Take care in asking for clarification, commenting, and answering.
Check out our Code of Conduct.











share|cite|improve this question









New contributor




b.ben is a new contributor to this site. Take care in asking for clarification, commenting, and answering.
Check out our Code of Conduct.









share|cite|improve this question




share|cite|improve this question








edited Jan 4 at 11:42









Blue

47.7k870151




47.7k870151






New contributor




b.ben is a new contributor to this site. Take care in asking for clarification, commenting, and answering.
Check out our Code of Conduct.









asked Jan 4 at 9:02









b.benb.ben

1063




1063




New contributor




b.ben is a new contributor to this site. Take care in asking for clarification, commenting, and answering.
Check out our Code of Conduct.





New contributor





b.ben is a new contributor to this site. Take care in asking for clarification, commenting, and answering.
Check out our Code of Conduct.






b.ben is a new contributor to this site. Take care in asking for clarification, commenting, and answering.
Check out our Code of Conduct.








  • 5




    A negative non-integer to a negative non-integer power is a little bit problematic (in real numbers) ...
    – Matti P.
    Jan 4 at 9:07






  • 1




    By the definition, if we wrote $x^x$ then the domain is $(0,+infty).$
    – Michael Rozenberg
    Jan 4 at 9:07












  • @MichaelRozenberg though if you want $x^{x^{x^{x^{...}}}}$ to converge then you need $e^{-e} leq x leq e^{1/e}$ roughly $0.066 lt x lt 1.44$
    – Henry
    Jan 4 at 9:14










  • @Henry I said about domain only.
    – Michael Rozenberg
    Jan 4 at 9:15










  • The domain of $y = x^x$ is $x > 0$ if you’re dealing with only real numbers.
    – KM101
    Jan 4 at 9:15
















  • 5




    A negative non-integer to a negative non-integer power is a little bit problematic (in real numbers) ...
    – Matti P.
    Jan 4 at 9:07






  • 1




    By the definition, if we wrote $x^x$ then the domain is $(0,+infty).$
    – Michael Rozenberg
    Jan 4 at 9:07












  • @MichaelRozenberg though if you want $x^{x^{x^{x^{...}}}}$ to converge then you need $e^{-e} leq x leq e^{1/e}$ roughly $0.066 lt x lt 1.44$
    – Henry
    Jan 4 at 9:14










  • @Henry I said about domain only.
    – Michael Rozenberg
    Jan 4 at 9:15










  • The domain of $y = x^x$ is $x > 0$ if you’re dealing with only real numbers.
    – KM101
    Jan 4 at 9:15










5




5




A negative non-integer to a negative non-integer power is a little bit problematic (in real numbers) ...
– Matti P.
Jan 4 at 9:07




A negative non-integer to a negative non-integer power is a little bit problematic (in real numbers) ...
– Matti P.
Jan 4 at 9:07




1




1




By the definition, if we wrote $x^x$ then the domain is $(0,+infty).$
– Michael Rozenberg
Jan 4 at 9:07






By the definition, if we wrote $x^x$ then the domain is $(0,+infty).$
– Michael Rozenberg
Jan 4 at 9:07














@MichaelRozenberg though if you want $x^{x^{x^{x^{...}}}}$ to converge then you need $e^{-e} leq x leq e^{1/e}$ roughly $0.066 lt x lt 1.44$
– Henry
Jan 4 at 9:14




@MichaelRozenberg though if you want $x^{x^{x^{x^{...}}}}$ to converge then you need $e^{-e} leq x leq e^{1/e}$ roughly $0.066 lt x lt 1.44$
– Henry
Jan 4 at 9:14












@Henry I said about domain only.
– Michael Rozenberg
Jan 4 at 9:15




@Henry I said about domain only.
– Michael Rozenberg
Jan 4 at 9:15












The domain of $y = x^x$ is $x > 0$ if you’re dealing with only real numbers.
– KM101
Jan 4 at 9:15






The domain of $y = x^x$ is $x > 0$ if you’re dealing with only real numbers.
– KM101
Jan 4 at 9:15












2 Answers
2






active

oldest

votes


















1














The tower is $lim_{ntoinfty}u_n$ with $u_1=x,,u_{n+1}=x^{u_n}$. If $x=sqrt{2}$ this is an increasing sequence of positive numbers $<2$. If $x=-sqrt{2}$, on the other hand, it quickly goes off the rails. What is $(-sqrt{2})^{-sqrt{2}}$ supposed to be?






share|cite|improve this answer





























    1














    You have $x^{x^{x^{dots}}} = 2$. You can simply substitute in the exponent to conclude that $x^2 = 2$. But that's only one way. If, instead you have that $x^2=2$, it's not necessarily true that $x^{x^{x^{dots}}} = 2$. So solving $x^2=2$ lets us find candidate solutions for the original equation, but then we need to confirm that each candidate is in fact a solution.



    First, let's check whether $x=sqrt{2}$ satisfy $x^{x^{x^{dots}}} = 2$. For this we need to know what $x^{x^{x^{dots}}} = 2$ actually is. We can define it as the limit of the sequence $x, x^x, x^{x^x}, x^{x^{x^x}}, dots$, and we can check that, if $x=sqrt{2}$, this sequence indeed converges to 2.



    What about $x=-sqrt{2}$? Consider $(-sqrt{2})^{-sqrt{2}}$. This is not even defined in real numbers. See How do you compute negative numbers to fractional powers? So we need to go to complex numbers. Then $(-sqrt{2})^{-sqrt{2}}$ actually takes infinitely many values, which causes problems on its own. EDIT: We can consider the sequence of principal values and try to check if it converges, but that turns out to be more complex than I thought.






    share|cite|improve this answer























      Your Answer





      StackExchange.ifUsing("editor", function () {
      return StackExchange.using("mathjaxEditing", function () {
      StackExchange.MarkdownEditor.creationCallbacks.add(function (editor, postfix) {
      StackExchange.mathjaxEditing.prepareWmdForMathJax(editor, postfix, [["$", "$"], ["\\(","\\)"]]);
      });
      });
      }, "mathjax-editing");

      StackExchange.ready(function() {
      var channelOptions = {
      tags: "".split(" "),
      id: "69"
      };
      initTagRenderer("".split(" "), "".split(" "), channelOptions);

      StackExchange.using("externalEditor", function() {
      // Have to fire editor after snippets, if snippets enabled
      if (StackExchange.settings.snippets.snippetsEnabled) {
      StackExchange.using("snippets", function() {
      createEditor();
      });
      }
      else {
      createEditor();
      }
      });

      function createEditor() {
      StackExchange.prepareEditor({
      heartbeatType: 'answer',
      autoActivateHeartbeat: false,
      convertImagesToLinks: true,
      noModals: true,
      showLowRepImageUploadWarning: true,
      reputationToPostImages: 10,
      bindNavPrevention: true,
      postfix: "",
      imageUploader: {
      brandingHtml: "Powered by u003ca class="icon-imgur-white" href="https://imgur.com/"u003eu003c/au003e",
      contentPolicyHtml: "User contributions licensed under u003ca href="https://creativecommons.org/licenses/by-sa/3.0/"u003ecc by-sa 3.0 with attribution requiredu003c/au003e u003ca href="https://stackoverflow.com/legal/content-policy"u003e(content policy)u003c/au003e",
      allowUrls: true
      },
      noCode: true, onDemand: true,
      discardSelector: ".discard-answer"
      ,immediatelyShowMarkdownHelp:true
      });


      }
      });






      b.ben is a new contributor. Be nice, and check out our Code of Conduct.










      draft saved

      draft discarded


















      StackExchange.ready(
      function () {
      StackExchange.openid.initPostLogin('.new-post-login', 'https%3a%2f%2fmath.stackexchange.com%2fquestions%2f3061444%2fin-xxxx-2-the-answer-is-x-sqrt2-why-not-also-sqrt2%23new-answer', 'question_page');
      }
      );

      Post as a guest















      Required, but never shown

























      2 Answers
      2






      active

      oldest

      votes








      2 Answers
      2






      active

      oldest

      votes









      active

      oldest

      votes






      active

      oldest

      votes









      1














      The tower is $lim_{ntoinfty}u_n$ with $u_1=x,,u_{n+1}=x^{u_n}$. If $x=sqrt{2}$ this is an increasing sequence of positive numbers $<2$. If $x=-sqrt{2}$, on the other hand, it quickly goes off the rails. What is $(-sqrt{2})^{-sqrt{2}}$ supposed to be?






      share|cite|improve this answer


























        1














        The tower is $lim_{ntoinfty}u_n$ with $u_1=x,,u_{n+1}=x^{u_n}$. If $x=sqrt{2}$ this is an increasing sequence of positive numbers $<2$. If $x=-sqrt{2}$, on the other hand, it quickly goes off the rails. What is $(-sqrt{2})^{-sqrt{2}}$ supposed to be?






        share|cite|improve this answer
























          1












          1








          1






          The tower is $lim_{ntoinfty}u_n$ with $u_1=x,,u_{n+1}=x^{u_n}$. If $x=sqrt{2}$ this is an increasing sequence of positive numbers $<2$. If $x=-sqrt{2}$, on the other hand, it quickly goes off the rails. What is $(-sqrt{2})^{-sqrt{2}}$ supposed to be?






          share|cite|improve this answer












          The tower is $lim_{ntoinfty}u_n$ with $u_1=x,,u_{n+1}=x^{u_n}$. If $x=sqrt{2}$ this is an increasing sequence of positive numbers $<2$. If $x=-sqrt{2}$, on the other hand, it quickly goes off the rails. What is $(-sqrt{2})^{-sqrt{2}}$ supposed to be?







          share|cite|improve this answer












          share|cite|improve this answer



          share|cite|improve this answer










          answered Jan 4 at 11:47









          J.G.J.G.

          23.3k22137




          23.3k22137























              1














              You have $x^{x^{x^{dots}}} = 2$. You can simply substitute in the exponent to conclude that $x^2 = 2$. But that's only one way. If, instead you have that $x^2=2$, it's not necessarily true that $x^{x^{x^{dots}}} = 2$. So solving $x^2=2$ lets us find candidate solutions for the original equation, but then we need to confirm that each candidate is in fact a solution.



              First, let's check whether $x=sqrt{2}$ satisfy $x^{x^{x^{dots}}} = 2$. For this we need to know what $x^{x^{x^{dots}}} = 2$ actually is. We can define it as the limit of the sequence $x, x^x, x^{x^x}, x^{x^{x^x}}, dots$, and we can check that, if $x=sqrt{2}$, this sequence indeed converges to 2.



              What about $x=-sqrt{2}$? Consider $(-sqrt{2})^{-sqrt{2}}$. This is not even defined in real numbers. See How do you compute negative numbers to fractional powers? So we need to go to complex numbers. Then $(-sqrt{2})^{-sqrt{2}}$ actually takes infinitely many values, which causes problems on its own. EDIT: We can consider the sequence of principal values and try to check if it converges, but that turns out to be more complex than I thought.






              share|cite|improve this answer




























                1














                You have $x^{x^{x^{dots}}} = 2$. You can simply substitute in the exponent to conclude that $x^2 = 2$. But that's only one way. If, instead you have that $x^2=2$, it's not necessarily true that $x^{x^{x^{dots}}} = 2$. So solving $x^2=2$ lets us find candidate solutions for the original equation, but then we need to confirm that each candidate is in fact a solution.



                First, let's check whether $x=sqrt{2}$ satisfy $x^{x^{x^{dots}}} = 2$. For this we need to know what $x^{x^{x^{dots}}} = 2$ actually is. We can define it as the limit of the sequence $x, x^x, x^{x^x}, x^{x^{x^x}}, dots$, and we can check that, if $x=sqrt{2}$, this sequence indeed converges to 2.



                What about $x=-sqrt{2}$? Consider $(-sqrt{2})^{-sqrt{2}}$. This is not even defined in real numbers. See How do you compute negative numbers to fractional powers? So we need to go to complex numbers. Then $(-sqrt{2})^{-sqrt{2}}$ actually takes infinitely many values, which causes problems on its own. EDIT: We can consider the sequence of principal values and try to check if it converges, but that turns out to be more complex than I thought.






                share|cite|improve this answer


























                  1












                  1








                  1






                  You have $x^{x^{x^{dots}}} = 2$. You can simply substitute in the exponent to conclude that $x^2 = 2$. But that's only one way. If, instead you have that $x^2=2$, it's not necessarily true that $x^{x^{x^{dots}}} = 2$. So solving $x^2=2$ lets us find candidate solutions for the original equation, but then we need to confirm that each candidate is in fact a solution.



                  First, let's check whether $x=sqrt{2}$ satisfy $x^{x^{x^{dots}}} = 2$. For this we need to know what $x^{x^{x^{dots}}} = 2$ actually is. We can define it as the limit of the sequence $x, x^x, x^{x^x}, x^{x^{x^x}}, dots$, and we can check that, if $x=sqrt{2}$, this sequence indeed converges to 2.



                  What about $x=-sqrt{2}$? Consider $(-sqrt{2})^{-sqrt{2}}$. This is not even defined in real numbers. See How do you compute negative numbers to fractional powers? So we need to go to complex numbers. Then $(-sqrt{2})^{-sqrt{2}}$ actually takes infinitely many values, which causes problems on its own. EDIT: We can consider the sequence of principal values and try to check if it converges, but that turns out to be more complex than I thought.






                  share|cite|improve this answer














                  You have $x^{x^{x^{dots}}} = 2$. You can simply substitute in the exponent to conclude that $x^2 = 2$. But that's only one way. If, instead you have that $x^2=2$, it's not necessarily true that $x^{x^{x^{dots}}} = 2$. So solving $x^2=2$ lets us find candidate solutions for the original equation, but then we need to confirm that each candidate is in fact a solution.



                  First, let's check whether $x=sqrt{2}$ satisfy $x^{x^{x^{dots}}} = 2$. For this we need to know what $x^{x^{x^{dots}}} = 2$ actually is. We can define it as the limit of the sequence $x, x^x, x^{x^x}, x^{x^{x^x}}, dots$, and we can check that, if $x=sqrt{2}$, this sequence indeed converges to 2.



                  What about $x=-sqrt{2}$? Consider $(-sqrt{2})^{-sqrt{2}}$. This is not even defined in real numbers. See How do you compute negative numbers to fractional powers? So we need to go to complex numbers. Then $(-sqrt{2})^{-sqrt{2}}$ actually takes infinitely many values, which causes problems on its own. EDIT: We can consider the sequence of principal values and try to check if it converges, but that turns out to be more complex than I thought.







                  share|cite|improve this answer














                  share|cite|improve this answer



                  share|cite|improve this answer








                  edited Jan 4 at 11:59

























                  answered Jan 4 at 9:22









                  Todor MarkovTodor Markov

                  1,619410




                  1,619410






















                      b.ben is a new contributor. Be nice, and check out our Code of Conduct.










                      draft saved

                      draft discarded


















                      b.ben is a new contributor. Be nice, and check out our Code of Conduct.













                      b.ben is a new contributor. Be nice, and check out our Code of Conduct.












                      b.ben is a new contributor. Be nice, and check out our Code of Conduct.
















                      Thanks for contributing an answer to Mathematics Stack Exchange!


                      • Please be sure to answer the question. Provide details and share your research!

                      But avoid



                      • Asking for help, clarification, or responding to other answers.

                      • Making statements based on opinion; back them up with references or personal experience.


                      Use MathJax to format equations. MathJax reference.


                      To learn more, see our tips on writing great answers.





                      Some of your past answers have not been well-received, and you're in danger of being blocked from answering.


                      Please pay close attention to the following guidance:


                      • Please be sure to answer the question. Provide details and share your research!

                      But avoid



                      • Asking for help, clarification, or responding to other answers.

                      • Making statements based on opinion; back them up with references or personal experience.


                      To learn more, see our tips on writing great answers.




                      draft saved


                      draft discarded














                      StackExchange.ready(
                      function () {
                      StackExchange.openid.initPostLogin('.new-post-login', 'https%3a%2f%2fmath.stackexchange.com%2fquestions%2f3061444%2fin-xxxx-2-the-answer-is-x-sqrt2-why-not-also-sqrt2%23new-answer', 'question_page');
                      }
                      );

                      Post as a guest















                      Required, but never shown





















































                      Required, but never shown














                      Required, but never shown












                      Required, but never shown







                      Required, but never shown

































                      Required, but never shown














                      Required, but never shown












                      Required, but never shown







                      Required, but never shown







                      Popular posts from this blog

                      An IMO inspired problem

                      Management

                      Investment